Đến nội dung

Hình ảnh

$\frac{b}{{{a^2}}} + \frac{c}{{{b^2}}} + \frac{a}{{{c^2}}} \geqslant 3\left( {\frac{1}{{{a^2}}} + \frac{1}{{{b^2}}} + \frac{1}{{{c^2}}}} \right)$


  • Please log in to reply
Chủ đề này có 3 trả lời

#1
hathu123

hathu123

    Lính mới

  • Thành viên mới
  • 7 Bài viết

Bài 1: Cho $a,b,c>0$ thỏa mãn: $ab+bc+ca=abc$. Chứng minh rằng: $$\frac{b}{{{a^2}}} + \frac{c}{{{b^2}}} + \frac{a}{{{c^2}}} \geqslant 3\left( {\frac{1}{{{a^2}}} + \frac{1}{{{b^2}}} + \frac{1}{{{c^2}}}} \right)$$Bài 2: Cho $a,b,c>0$. Chứng minh rằng: $$\frac{{{{\left( {a + b} \right)}^2}}}{{{c^2} + ab}} + \frac{{{{\left( {b + c} \right)}^2}}}{{{a^2} + bc}} + \frac{{{{\left( {c + a} \right)}^2}}}{{{b^2} + ac}} \geqslant 6$$



#2
tuaneee111

tuaneee111

    Trung sĩ

  • Thành viên
  • 174 Bài viết

Bài 1:

Đặt: $\left( {\frac{1}{a};\frac{1}{b};\frac{1}{c}} \right) \to \left( {x;y;z} \right) \Rightarrow x + y + z = 1 \Rightarrow {\left( {x + y + z} \right)^2} = 1$

Bất đẳng thức trở thành: $$\frac{{{x^2}}}{y} + \frac{{{y^2}}}{z} + \frac{{{z^2}}}{x} \geqslant 3\left( {{x^2} + {y^2} + {z^2}} \right)$$Sử dụng giả thiết ta có: $$\frac{{{x^2}}}{y} + \frac{{{y^2}}}{z} + \frac{{{z^2}}}{x} - 3\left( {{x^2} + {y^2} + {z^2}} \right) = \sum\limits_{cyc} {{{\left( {x - y} \right)}^2}\left( {\frac{1}{y} - 1} \right)}  = \sum\limits_{cyc} {{{\left( {x - y} \right)}^2}\left( {\frac{{x + z}}{y}} \right) \geqslant 0 \Rightarrow Q.E.D} $$


Bài viết đã được chỉnh sửa nội dung bởi tuaneee111: 09-06-2017 - 18:31

$$\boxed{\boxed{I\heartsuit MATHEMATICAL}}$$

Blog của tôi

:luoi: Sức hấp dẫn của toán học mãnh liệt đến nỗi tôi bắt đầu sao nhãng các môn học khác - Sofia Vasilyevna Kovalevskaya :lol:


#3
Hoang Dinh Nhat

Hoang Dinh Nhat

    Sĩ quan

  • Thành viên
  • 402 Bài viết

Bài 2: Vì BĐT thuần nhất nên chuẩn hóa: $a+b+c=3$ ($0<a,b,c<3$)

Theo AM-GM ta có: $\sum \frac{(a+b)^2}{c^2+ab}\geq \sum \frac{4(3-c)^2}{4c^2+(3-c)^2}$

Có đánh giá: $\frac{4(3-c)^2}{4c^2+(3-c)^2}\geq -3c+5$$\Leftrightarrow 3(x-1)^2(5x-3)\geq 0$

TH1: Nếu $3>a,b,c\geq \frac{3}{5}$ thì đánh giá trên luôn đúng nên áp dụng BĐT trên, ta có: $\sum \frac{(a+b)^2}{c^2+ab}\geq \sum \frac{4(3-c)^2}{4c^2+(3-c)^2}\geq -3(a+b+c)+15=6$

TH2: Nếu $0<a,b,c< \frac{3}{5}$ thì dễ dàng cm được $\frac{(a+b)^2}{c^2+ab}>2$ nên $\sum \frac{(a+b)^2}{c^2+ab}>6$

Do đó: $\sum \frac{(a+b)^2}{c^2+ab}\geq 6$

Đạt tại: $a=b=c$


Bài viết đã được chỉnh sửa nội dung bởi Hoang Dinh Nhat: 09-06-2017 - 18:39

Chấp nhận giới hạn của bản thân, nhưng đừng bao giờ bỏ cuộc

 

 

 

 


#4
tuaneee111

tuaneee111

    Trung sĩ

  • Thành viên
  • 174 Bài viết

Bài 2.

Không mất tính tổng quát ta giả sử: $a \geqslant b \geqslant c$

Ta có: $$\frac{{{{\left( {a + b} \right)}^2}}}{{{c^2} + ab}} + \frac{{{{\left( {b + c} \right)}^2}}}{{{a^2} + bc}} + \frac{{{{\left( {c + a} \right)}^2}}}{{{b^2} + ac}} - 6 = \sum\limits_{cyc} {{{\left( {a - c} \right)}^2}\frac{{\left( {a + c} \right)\left( {a + c - b} \right)}}{{\left( {{c^2} + ab} \right)\left( {{a^2} + bc} \right)}}} $$Đăt: $${{S_a} = \frac{{\left( {b + c} \right)\left( {b + c - a} \right)}}{{\left( {{b^2} + ac} \right)\left( {{c^2} + ab} \right)}};\,\,{S_b} = \frac{{\left( {a + c} \right)\left( {a + c - b} \right)}}{{\left( {{c^2} + ab} \right)\left( {{a^2} + bc} \right)}};\,\,{S_c} = \frac{{\left( {a + b} \right)\left( {a + b - c} \right)}}{{\left( {{a^2} + bc} \right)\left( {{b^2} + ca} \right)}}}$$Do $a \geqslant b \geqslant c \Rightarrow {S_b} > 0;\,\,{S_c} > 0$. Bất đẳng thức đúng nếu ta chứng minh được: ${b^2}{S_a} + {a^2}{S_b} \geqslant 0$. Ta có: $${b^2}{S_a} + {a^2}{S_b} = \frac{{{b^2}\left( {b + c} \right)\left( {b + c - a} \right)}}{{{b^2} + ac}} + \frac{{{a^2}\left( {a + c} \right)\left( {a + c - b} \right)}}{{{a^2} + bc}}$$$$ \geqslant \left( {b + c} \right)\left[ {\frac{{{b^2}\left( {b + c - a} \right)}}{{{b^2} + ac}} + \frac{{{a^2}\left( {a + c - b} \right)}}{{{a^2} + bc}}} \right]$$$$ = \frac{{\left( {b + c} \right)\left( {{a^2}{b^2}c + {a^4}c + {a^3}{c^2} - {a^3}bc + {a^2}{b^2}c + {b^4}c - {b^3}ac} \right)}}{{\left( {{b^2} + ac} \right)\left( {{a^2} + bc} \right)}}$$Theo $AM-GM$ ta có: $ \displaystyle \left\{ \begin{array}{l}{{a}^{2}}{{b}^{2}}c+{{a}^{4}}c\ge 2{{a}^{3}}bc>{{a}^{3}}bc\\{{a}^{2}}{{b}^{2}}c+{{b}^{4}}c\ge 2{{b}^{3}}ac>{{b}^{3}}ac\end{array} \right.\Rightarrow {{b}^{2}}{{S}_{a}}+{{a}^{2}}{{S}_{b}}>0$

Vậy có đpcm!


$$\boxed{\boxed{I\heartsuit MATHEMATICAL}}$$

Blog của tôi

:luoi: Sức hấp dẫn của toán học mãnh liệt đến nỗi tôi bắt đầu sao nhãng các môn học khác - Sofia Vasilyevna Kovalevskaya :lol:





0 người đang xem chủ đề

0 thành viên, 0 khách, 0 thành viên ẩn danh